Showing something satisfies Inner Product - Involves Orthogonal Matrices

Click For Summary
SUMMARY

This discussion focuses on proving that the product A = x · Ay constitutes an inner product for R3, where A is defined as A = Z-1DZ with Z being a 3×3 orthogonal matrix and D a diagonal matrix containing positive integers. The key steps involve recognizing that Z-1 equals Z^T and leveraging the properties of orthogonal matrices to demonstrate that ≥ 0. The participants emphasize the importance of the positive definiteness of matrix D and the transformation properties of orthogonal matrices in the proof.

PREREQUISITES
  • Understanding of orthogonal matrices and their properties
  • Familiarity with inner product spaces and definitions
  • Knowledge of diagonal matrices and their characteristics
  • Basic linear algebra concepts, particularly matrix multiplication and transposition
NEXT STEPS
  • Study the properties of orthogonal matrices in linear algebra
  • Learn about positive definite matrices and their implications in inner product spaces
  • Explore the relationship between matrix products and inner products in R3
  • Investigate the role of diagonal matrices in transformations and their effects on vector spaces
USEFUL FOR

Students and educators in linear algebra, mathematicians focusing on inner product spaces, and anyone interested in the applications of orthogonal matrices in mathematical proofs.

Circular_Block
Messages
2
Reaction score
0

Homework Statement


[/B]
Let Z be any 3×3 orthogonal matrix and let A = Z-1DZ where D is a diagonal matrix with positive integers along its diagonal.
Show that the product <x, y> A = x · Ay is an inner product for R3.

Homework Equations


None

The Attempt at a Solution



I've shown that x · Dy is an inner product. I know that Z-1 is equal to ZT. I believe that will lead me somewhere. I'm just having trouble showing the property <x, x> ≥ 0. I also know that (ATx)⋅x = x ⋅ Ax.

Just missing one step. I don't know what it is.
 
Last edited:
Physics news on Phys.org
Hint: ## (Zx)^T = x^T Z^T ##
 
Doing that you'll end up with (ATx)⋅x right?
 
Last edited:
Circular_Block said:
Doing that you'll end up with (ATx)⋅x right?

x.y=x^Ty. That turns a dot product into a matrix product. Add that to the list of clues.
 
Don't neglect that D is all positive. So ##Z^T D Z## should also be non-negative, right?
 
Question: A clock's minute hand has length 4 and its hour hand has length 3. What is the distance between the tips at the moment when it is increasing most rapidly?(Putnam Exam Question) Answer: Making assumption that both the hands moves at constant angular velocities, the answer is ## \sqrt{7} .## But don't you think this assumption is somewhat doubtful and wrong?

Similar threads

  • · Replies 5 ·
Replies
5
Views
4K
  • · Replies 2 ·
Replies
2
Views
2K
  • · Replies 11 ·
Replies
11
Views
3K
  • · Replies 9 ·
Replies
9
Views
2K
Replies
4
Views
3K
  • · Replies 6 ·
Replies
6
Views
2K
Replies
11
Views
4K
  • · Replies 6 ·
Replies
6
Views
2K
  • · Replies 4 ·
Replies
4
Views
2K
Replies
4
Views
2K